simple question about properly divergent sequence












2












$begingroup$


Is it true that if a sequence ${x_{n}}$ has no properly divergent subsequence, then the sequence is bounded?



I think it is true intuitively considering subsequential limit but i’m not sure about that.



Give me some counter-example or comment. Thank you.










share|cite|improve this question









$endgroup$








  • 1




    $begingroup$
    If the sequence is not bounded then either there is a subsequence strictly increasing to $infty$ or there is a subsequence strictly decreasing to $-infty$
    $endgroup$
    – Kavi Rama Murthy
    Jan 3 at 6:33










  • $begingroup$
    Properly divergent sequence? What is the meaning of this, please?
    $endgroup$
    – dmtri
    Jan 3 at 6:49










  • $begingroup$
    @KaviRamaMurthy Actually, I tried to show your statement in another way using the assumption as above leads a contradiction.
    $endgroup$
    – Primavera
    Jan 4 at 7:25










  • $begingroup$
    @dmtri diverges at plus/minus infinity not oscillate
    $endgroup$
    – Primavera
    Jan 4 at 7:26
















2












$begingroup$


Is it true that if a sequence ${x_{n}}$ has no properly divergent subsequence, then the sequence is bounded?



I think it is true intuitively considering subsequential limit but i’m not sure about that.



Give me some counter-example or comment. Thank you.










share|cite|improve this question









$endgroup$








  • 1




    $begingroup$
    If the sequence is not bounded then either there is a subsequence strictly increasing to $infty$ or there is a subsequence strictly decreasing to $-infty$
    $endgroup$
    – Kavi Rama Murthy
    Jan 3 at 6:33










  • $begingroup$
    Properly divergent sequence? What is the meaning of this, please?
    $endgroup$
    – dmtri
    Jan 3 at 6:49










  • $begingroup$
    @KaviRamaMurthy Actually, I tried to show your statement in another way using the assumption as above leads a contradiction.
    $endgroup$
    – Primavera
    Jan 4 at 7:25










  • $begingroup$
    @dmtri diverges at plus/minus infinity not oscillate
    $endgroup$
    – Primavera
    Jan 4 at 7:26














2












2








2





$begingroup$


Is it true that if a sequence ${x_{n}}$ has no properly divergent subsequence, then the sequence is bounded?



I think it is true intuitively considering subsequential limit but i’m not sure about that.



Give me some counter-example or comment. Thank you.










share|cite|improve this question









$endgroup$




Is it true that if a sequence ${x_{n}}$ has no properly divergent subsequence, then the sequence is bounded?



I think it is true intuitively considering subsequential limit but i’m not sure about that.



Give me some counter-example or comment. Thank you.







real-analysis sequences-and-series






share|cite|improve this question













share|cite|improve this question











share|cite|improve this question




share|cite|improve this question










asked Jan 3 at 6:31









PrimaveraPrimavera

31619




31619








  • 1




    $begingroup$
    If the sequence is not bounded then either there is a subsequence strictly increasing to $infty$ or there is a subsequence strictly decreasing to $-infty$
    $endgroup$
    – Kavi Rama Murthy
    Jan 3 at 6:33










  • $begingroup$
    Properly divergent sequence? What is the meaning of this, please?
    $endgroup$
    – dmtri
    Jan 3 at 6:49










  • $begingroup$
    @KaviRamaMurthy Actually, I tried to show your statement in another way using the assumption as above leads a contradiction.
    $endgroup$
    – Primavera
    Jan 4 at 7:25










  • $begingroup$
    @dmtri diverges at plus/minus infinity not oscillate
    $endgroup$
    – Primavera
    Jan 4 at 7:26














  • 1




    $begingroup$
    If the sequence is not bounded then either there is a subsequence strictly increasing to $infty$ or there is a subsequence strictly decreasing to $-infty$
    $endgroup$
    – Kavi Rama Murthy
    Jan 3 at 6:33










  • $begingroup$
    Properly divergent sequence? What is the meaning of this, please?
    $endgroup$
    – dmtri
    Jan 3 at 6:49










  • $begingroup$
    @KaviRamaMurthy Actually, I tried to show your statement in another way using the assumption as above leads a contradiction.
    $endgroup$
    – Primavera
    Jan 4 at 7:25










  • $begingroup$
    @dmtri diverges at plus/minus infinity not oscillate
    $endgroup$
    – Primavera
    Jan 4 at 7:26








1




1




$begingroup$
If the sequence is not bounded then either there is a subsequence strictly increasing to $infty$ or there is a subsequence strictly decreasing to $-infty$
$endgroup$
– Kavi Rama Murthy
Jan 3 at 6:33




$begingroup$
If the sequence is not bounded then either there is a subsequence strictly increasing to $infty$ or there is a subsequence strictly decreasing to $-infty$
$endgroup$
– Kavi Rama Murthy
Jan 3 at 6:33












$begingroup$
Properly divergent sequence? What is the meaning of this, please?
$endgroup$
– dmtri
Jan 3 at 6:49




$begingroup$
Properly divergent sequence? What is the meaning of this, please?
$endgroup$
– dmtri
Jan 3 at 6:49












$begingroup$
@KaviRamaMurthy Actually, I tried to show your statement in another way using the assumption as above leads a contradiction.
$endgroup$
– Primavera
Jan 4 at 7:25




$begingroup$
@KaviRamaMurthy Actually, I tried to show your statement in another way using the assumption as above leads a contradiction.
$endgroup$
– Primavera
Jan 4 at 7:25












$begingroup$
@dmtri diverges at plus/minus infinity not oscillate
$endgroup$
– Primavera
Jan 4 at 7:26




$begingroup$
@dmtri diverges at plus/minus infinity not oscillate
$endgroup$
– Primavera
Jan 4 at 7:26










1 Answer
1






active

oldest

votes


















1












$begingroup$

Equivalently, if a sequence is unbounded, it has a sub-sequence strictly increasing to $infty$ or strictly decreasing to $-infty.$



Let $(x_j)_{jin Bbb N}$ be an unbounded sequence in $Bbb R$.



Let $n_1$ be the least $nin Bbb N$ such that $|x_n|>1.$ Define $n_{j+1}$ recursively as follows: Let $$y_j=max (j+1, max { |x_i|:1leq ileq n_j}).$$ Let $n_{j+1}$ be the least $nin Bbb N$ such that $|a_n|>y_j.$



For every $jin Bbb N$ we have $jleq n_j<n_{j+1}$ and $j<|x_{n_j}|<|x_{n_{j+1}}|.$



Let $S={jin Bbb N: x_{n_j}>j}$ and $T={jin Bbb N: x_{n_j}<-j}.$ We have $Scup T=Bbb N.$



If $S$ is an infinite set then the sequence $(x_{n_j})_{jin S}$ strictly increases to $infty.$



If $T$ is an infinite set then $(x_{n_j})_{jin T}$ strictly decreases to $-infty.$






share|cite|improve this answer











$endgroup$














    Your Answer





    StackExchange.ifUsing("editor", function () {
    return StackExchange.using("mathjaxEditing", function () {
    StackExchange.MarkdownEditor.creationCallbacks.add(function (editor, postfix) {
    StackExchange.mathjaxEditing.prepareWmdForMathJax(editor, postfix, [["$", "$"], ["\\(","\\)"]]);
    });
    });
    }, "mathjax-editing");

    StackExchange.ready(function() {
    var channelOptions = {
    tags: "".split(" "),
    id: "69"
    };
    initTagRenderer("".split(" "), "".split(" "), channelOptions);

    StackExchange.using("externalEditor", function() {
    // Have to fire editor after snippets, if snippets enabled
    if (StackExchange.settings.snippets.snippetsEnabled) {
    StackExchange.using("snippets", function() {
    createEditor();
    });
    }
    else {
    createEditor();
    }
    });

    function createEditor() {
    StackExchange.prepareEditor({
    heartbeatType: 'answer',
    autoActivateHeartbeat: false,
    convertImagesToLinks: true,
    noModals: true,
    showLowRepImageUploadWarning: true,
    reputationToPostImages: 10,
    bindNavPrevention: true,
    postfix: "",
    imageUploader: {
    brandingHtml: "Powered by u003ca class="icon-imgur-white" href="https://imgur.com/"u003eu003c/au003e",
    contentPolicyHtml: "User contributions licensed under u003ca href="https://creativecommons.org/licenses/by-sa/3.0/"u003ecc by-sa 3.0 with attribution requiredu003c/au003e u003ca href="https://stackoverflow.com/legal/content-policy"u003e(content policy)u003c/au003e",
    allowUrls: true
    },
    noCode: true, onDemand: true,
    discardSelector: ".discard-answer"
    ,immediatelyShowMarkdownHelp:true
    });


    }
    });














    draft saved

    draft discarded


















    StackExchange.ready(
    function () {
    StackExchange.openid.initPostLogin('.new-post-login', 'https%3a%2f%2fmath.stackexchange.com%2fquestions%2f3060306%2fsimple-question-about-properly-divergent-sequence%23new-answer', 'question_page');
    }
    );

    Post as a guest















    Required, but never shown

























    1 Answer
    1






    active

    oldest

    votes








    1 Answer
    1






    active

    oldest

    votes









    active

    oldest

    votes






    active

    oldest

    votes









    1












    $begingroup$

    Equivalently, if a sequence is unbounded, it has a sub-sequence strictly increasing to $infty$ or strictly decreasing to $-infty.$



    Let $(x_j)_{jin Bbb N}$ be an unbounded sequence in $Bbb R$.



    Let $n_1$ be the least $nin Bbb N$ such that $|x_n|>1.$ Define $n_{j+1}$ recursively as follows: Let $$y_j=max (j+1, max { |x_i|:1leq ileq n_j}).$$ Let $n_{j+1}$ be the least $nin Bbb N$ such that $|a_n|>y_j.$



    For every $jin Bbb N$ we have $jleq n_j<n_{j+1}$ and $j<|x_{n_j}|<|x_{n_{j+1}}|.$



    Let $S={jin Bbb N: x_{n_j}>j}$ and $T={jin Bbb N: x_{n_j}<-j}.$ We have $Scup T=Bbb N.$



    If $S$ is an infinite set then the sequence $(x_{n_j})_{jin S}$ strictly increases to $infty.$



    If $T$ is an infinite set then $(x_{n_j})_{jin T}$ strictly decreases to $-infty.$






    share|cite|improve this answer











    $endgroup$


















      1












      $begingroup$

      Equivalently, if a sequence is unbounded, it has a sub-sequence strictly increasing to $infty$ or strictly decreasing to $-infty.$



      Let $(x_j)_{jin Bbb N}$ be an unbounded sequence in $Bbb R$.



      Let $n_1$ be the least $nin Bbb N$ such that $|x_n|>1.$ Define $n_{j+1}$ recursively as follows: Let $$y_j=max (j+1, max { |x_i|:1leq ileq n_j}).$$ Let $n_{j+1}$ be the least $nin Bbb N$ such that $|a_n|>y_j.$



      For every $jin Bbb N$ we have $jleq n_j<n_{j+1}$ and $j<|x_{n_j}|<|x_{n_{j+1}}|.$



      Let $S={jin Bbb N: x_{n_j}>j}$ and $T={jin Bbb N: x_{n_j}<-j}.$ We have $Scup T=Bbb N.$



      If $S$ is an infinite set then the sequence $(x_{n_j})_{jin S}$ strictly increases to $infty.$



      If $T$ is an infinite set then $(x_{n_j})_{jin T}$ strictly decreases to $-infty.$






      share|cite|improve this answer











      $endgroup$
















        1












        1








        1





        $begingroup$

        Equivalently, if a sequence is unbounded, it has a sub-sequence strictly increasing to $infty$ or strictly decreasing to $-infty.$



        Let $(x_j)_{jin Bbb N}$ be an unbounded sequence in $Bbb R$.



        Let $n_1$ be the least $nin Bbb N$ such that $|x_n|>1.$ Define $n_{j+1}$ recursively as follows: Let $$y_j=max (j+1, max { |x_i|:1leq ileq n_j}).$$ Let $n_{j+1}$ be the least $nin Bbb N$ such that $|a_n|>y_j.$



        For every $jin Bbb N$ we have $jleq n_j<n_{j+1}$ and $j<|x_{n_j}|<|x_{n_{j+1}}|.$



        Let $S={jin Bbb N: x_{n_j}>j}$ and $T={jin Bbb N: x_{n_j}<-j}.$ We have $Scup T=Bbb N.$



        If $S$ is an infinite set then the sequence $(x_{n_j})_{jin S}$ strictly increases to $infty.$



        If $T$ is an infinite set then $(x_{n_j})_{jin T}$ strictly decreases to $-infty.$






        share|cite|improve this answer











        $endgroup$



        Equivalently, if a sequence is unbounded, it has a sub-sequence strictly increasing to $infty$ or strictly decreasing to $-infty.$



        Let $(x_j)_{jin Bbb N}$ be an unbounded sequence in $Bbb R$.



        Let $n_1$ be the least $nin Bbb N$ such that $|x_n|>1.$ Define $n_{j+1}$ recursively as follows: Let $$y_j=max (j+1, max { |x_i|:1leq ileq n_j}).$$ Let $n_{j+1}$ be the least $nin Bbb N$ such that $|a_n|>y_j.$



        For every $jin Bbb N$ we have $jleq n_j<n_{j+1}$ and $j<|x_{n_j}|<|x_{n_{j+1}}|.$



        Let $S={jin Bbb N: x_{n_j}>j}$ and $T={jin Bbb N: x_{n_j}<-j}.$ We have $Scup T=Bbb N.$



        If $S$ is an infinite set then the sequence $(x_{n_j})_{jin S}$ strictly increases to $infty.$



        If $T$ is an infinite set then $(x_{n_j})_{jin T}$ strictly decreases to $-infty.$







        share|cite|improve this answer














        share|cite|improve this answer



        share|cite|improve this answer








        edited Jan 3 at 9:15

























        answered Jan 3 at 9:10









        DanielWainfleetDanielWainfleet

        35.7k31648




        35.7k31648






























            draft saved

            draft discarded




















































            Thanks for contributing an answer to Mathematics Stack Exchange!


            • Please be sure to answer the question. Provide details and share your research!

            But avoid



            • Asking for help, clarification, or responding to other answers.

            • Making statements based on opinion; back them up with references or personal experience.


            Use MathJax to format equations. MathJax reference.


            To learn more, see our tips on writing great answers.




            draft saved


            draft discarded














            StackExchange.ready(
            function () {
            StackExchange.openid.initPostLogin('.new-post-login', 'https%3a%2f%2fmath.stackexchange.com%2fquestions%2f3060306%2fsimple-question-about-properly-divergent-sequence%23new-answer', 'question_page');
            }
            );

            Post as a guest















            Required, but never shown





















































            Required, but never shown














            Required, but never shown












            Required, but never shown







            Required, but never shown

































            Required, but never shown














            Required, but never shown












            Required, but never shown







            Required, but never shown







            Popular posts from this blog

            To store a contact into the json file from server.js file using a class in NodeJS

            Redirect URL with Chrome Remote Debugging Android Devices

            Dieringhausen